Difference between revisions of "2023 AMC 8 Problems/Problem 11"

(Written Solution)
Line 1: Line 1:
 
==Written Solution==
 
==Written Solution==
 
+
Since the answers are so far appart, we just need to compute the # of figures the answer contains. So by approximating all the values for the hourly rate we have <math>\frac{300,000,000}{5 * 30 * 30} \approx \frac{300,000,000}{5000} = 60,000</math> which is <math>\boxed{\text{(C)}60,000}</math>  
<math>\frac{292,526,838}{6.5} \approx \boxed{\text{(C)}60,000}</math>. This is the answer because <math>60,000</math> is closer to <math>\frac{292,526,838}{6.5}</math> than to all the other options.
 
  
 
~apex304, SohumUttamchandani, wuwang2002, TaeKim, Cxrupptedpat
 
~apex304, SohumUttamchandani, wuwang2002, TaeKim, Cxrupptedpat
 
  
 
==Animated Video Solution==
 
==Animated Video Solution==

Revision as of 20:55, 24 January 2023

Written Solution

Since the answers are so far appart, we just need to compute the # of figures the answer contains. So by approximating all the values for the hourly rate we have $\frac{300,000,000}{5 * 30 * 30} \approx \frac{300,000,000}{5000} = 60,000$ which is $\boxed{\text{(C)}60,000}$

~apex304, SohumUttamchandani, wuwang2002, TaeKim, Cxrupptedpat

Animated Video Solution

https://youtu.be/hwR2VM9tHJ0

~Star League (https://starleague.us)